Solution to Exercise of
Polynomial Methods in Combinatorics
Solution 2.1.
We can show the general case directly.
Claim. Given any -planes in and there exists a non-zero polynomial of degree that vanishes on all the -planes.
Notice that, if a polynomial with degree of vanishes at points on a same -plane, then it will vanish on this -plane. Therefore, there exists a non-zero polynomial satisfying the condition when , that is , which means . ∎
Solution 2.2.
Since the general process of proving the lemma using the Vandermonde determinant has been given in the question, we omit the proof using this method and instead use the proof of constructing the polynomial .
Define and , then we have if and only if so that we know is injective. Further, is an isomorphism.
Regarding the implication relation between the two lemmas, if vanishes on every point in , that is, for every , we have , which implies . ∎
Solution 2.3.
We will use induction to prove this lemma.
If , we know have at most zero points. Now we proceed by induction on . We can rewrite as . And we know , so we have by induction hypothesis. ∎
Solution 2.4.
11 1 This part of the proof comes from [3].We will use induction to prove this conclusion. Suppose this conclusion is established for polynomials in variables. We can rewrite , where . Notice that, if , then must satisfy at least one condition:
(1) For any ,
(2) Fix , then (Notice that ).
First, We assume and . By inductive hypothesis, we know . For any fixed , we know there only exist at most roots of by fundamental theorem of algebra. Now applying Fubini’s theorem then we have , which leads . ∎
Solution 2.5.
We can take . If this conclusion isn’t established, assuming , we can find that vanishes at all points in with because
We can also assume that won’t exceed (if not, we can assume as to replace ). Further, we have and if assuming .
Claim. If that vanishes at all points in , then we have and also vanishes at all points in .
Because of for any , holds for any , that is, . If we assume , we have . Now take and we know for any . Further, we know for any , that is, , which means also vanishes at all points in .
If we assume that is the polynomial with the lowest degree satisfying that vanishes at all points in , we have , implying , which causes the contradiction. ∎
Solution 2.6.
Assuming the maximum number of joints generated by lines is , then we have the following claim.
Claim. There exists one line with at most joints.
First we know we can find a non-zero polynomial with degree at most vanishing at every joint. (Let be the polynomial with the lowest degree that satisfies this condition.) If every line contains over joints, then vanishes on every line. So we know also vanish on every joint. Due to the minimum degree of , we have , which leads a contradiction.
Now we can know from
∎
Solution 2.7.
We assume that set contains all the joints. For each coordinate axis , define the indicator function as:
Then we know and the cardinality of can be expressed as:
For non-negative functions , Hölder’s inequality states:
Notice that , so we have , which finishes the proof. ∎
Solution 2.8.
It’s trivial that vanishes at all points. If there exist a polynomial with , we can rewrite it as and get for any , from vanishing at points when and are fixed. And we can rewrite as . Similarly, we know for any . Then we have , which leads a contradiction.
Now we know that every minimal degree polynomial vanishing on the grid has degree of . Consider that for every fixed and , we have . Then must be form of . Considering the degree, we know is a constant, that is, is a multiple of .
For the case of , we will prove this conclusion: for every minimal degree polynomial vanishing on the grid, must be the form of
Assuming , we can rewrite
where satisfying:
(1) Its degree is at most ;
(2) None of its monomials contains the th power of a single variable;
(3) It vanishes on any combination where each of its variables is drawn from a fixed set of distinct points.
Claim. Any polynomial satisfying these three properties must be the zero polynomial.
We will prove this conclusion by induction on the number of variables. It’s obvious that the conclusion establish when because we know it has degree at most from condition (1) and (2). Suppose that the conclusion holds for polynomials with variables. Let satisfy the three conditions and vanish on where . Now we can rewrite as , then we know by and condition (3). Then we know that vanishes on and also satisfies condition (1) and (2), for any . Now we can know by induction hypothesis, which finishes the proof.
Finally, let and the proof of the conclusion is completed. ∎
Solution 2.9.
The conclusion of this exercise can be fully supported by the proof presented in [8]. ∎
Solution 3.1.
Firstly, we can define in as , which equals to , so every point on the Hermitian variety satisfies .
Claim. For any , there exist satisfying .
For any , we can extend it to an orthonormal basis satisfying
Also, we can construct another orthonormal basis satisfying
Then let , it’s obvious that keeps the inner product, which implies . Now we know this group action is transitively. ∎
Solution 3.2.
Firstly, about the number of points on hypersurface , we can rewrite the equation as and let . By results in [6], we know the equation has solutions for if is fixed. Allow and to vary, then we know the number of the solutions is , which means there are points on .
Secondly, about the number of lines lying on , we need the following claim.
Claim. Each point of lies on lines.
Consider that always holds for every , it implies and . Let and it’s obvious that contains points for fixed by [6]. Notice that are linear for , so the number of points on satisfying the above linear equation is . Allow to vary from to , we know the number of solutions for of initial equations is . And because the above equations which contain are all homogeneous, there are only different directions, that is, each point corresponds to lines.
Because each point contributes lines and there are points on , each line is shared by points. Now we know there are lines on .
Finally, regarding of fact that the lines through each point lie in a 3 -plane, we can know every line passing is in , which is a 3 -plane, by the knowledge of linear algebra. ∎
Solution 3.3.
For the case of , we can consider the linear map . If , then there exist linear independent functions , and we know by Rank-Nullity Theorem, which leads . So there exists a non-zero function satisfying , which is a contradiction.
When , we assume that the case of is established. If regarding as a function with respect to , we have
where (the reason why we can make this decomposition is that forms a group of basis). Notice that satisfies vanishing lemma of degree , we have
according to the induction hypothesis. ∎
Solution 6.1.
We use the Bezout theorem in [4] to prove the following claim.
Claim. If , then the theorem is still true when is finite.
Firstly, we know
from [4]. We consider and as algebraic curve defined in and here . So for every line in initial , it’s also in the ”new ”.
Let lines be and now
Because are lines on and , and hold. So
which means
Notice that the lines in initial are all in then the claim is true. ∎
Solution 6.2.
For the first question, parameterize curve as and define polynomials
here are varible. Because and , we have and . So resultant is polynomial wrt and let . When , we know and have common root , then the resultant equals to 0. So holds for every , which means . Consider the form of resultant (a determinant), we know only appear in the right columns, so .
For the second question, firstly parameterize :
and here , then let
Let and , here are polynomials and , .
Now let . Notice that and are all like and . Similar as the first proof, just consider the determinant, we know because there are only columns containing .
For the general case, if is a polynomial map where is a curve or (hyper)surface with dimension of and degree of , and every is not more than , then there exist such that and . The details of proof can be read here: [1]. ∎
Solution 7.1.
33 3 Hint of this problem may has defect. This solution has fixed that.Using the hint of exercise, for an integer , let be the set of pairs with , and so that divides both and . So we have .
Note that can be formed by starting with all pairs with and then removing for every prime . Therefore,
When , we have , which means we only need prove that . And we can use to finish this proof. When , the ratios are respectively. So we have . ∎
Solution 7.2.
Let be the set of fractions in whose numerator and denominator are both at least . We will state that . We only need to prove that there are no more than fractions whose numerator or denominator is less than . Notice that (just consider for odd ), then every fraction in has a denominator less than . So there are at most fractions with numerator greater than and at most fractions with denominator greater than . Sum them up then we know the conclusion is true.
Let be the lines with slope in . Then every line with slope of in intersects the grid at most points. But every point in lies in at least lines of . By double counting, we know , so the proof finishes. ∎
Solution 7.3.
Every line in can be written as where represents point on the line and . Then line intersects line at only . Also, for a fixed point in , because can take different values (including ), there are different lines in passing it. Then we know immediately. ∎
Solution 7.4.
With some simple calculation, we know . For every satisfying and , we have passing , for (notice that ), which means every is an -rich point. So . ∎
Solution 7.5.
Take in Theorem 7.11 and then we have . Also, we have trivial bound of (for every point, only consider the lines passing through it). Now we have . Because , we have .
If , which implies , we have , that is, .
If , which implies , we have , that is, .
Then we have , which is Theorem 7.1. ∎
Solution 7.6.
Using and we construct a graph drawn in the plane. The vertices of our graph are the points of . We join two vertices with an edge of if the two points are two consecutive points of along a line . We let denote the number of points on . Notice that and , we have . The crossing number of this drawing is at most , since each crossing of the graph must correspond to an intersection of two lines of . This is also a straight line drawing. So we see that .
Claim. We have .
If , we can apply the estimate , then we get , which means . If , the conclusion holds naturally.
Now we know immediately. ∎
Solution 7.7.
Proof by Theorem 7.11: Take in Theorem 7.11 and then we have . Also, we have trivial bound of (for every line, only consider the points lying on it). Now we have . Because , we have . If , which implies , we have , that is, . If , which implies , we have , that is, . So the conclusion is established.
Proof by Theorem 7.1: Consider the polarity transformation
where , assuming both don’t contain or pass origin point, we have is a set of lines and is a set of points. And for every point , we have passing . For every line , we have lying on . By Theorem 7.1, we know . We get the conclusion since
∎
Solution 7.8.
Let , there must exist such that for (just let not equal to the root of for every ). So the projective transformation
can map to a set in . Because projective transformation don’t change the number of lines, points and incidences, directly, we know that Szemeredi-Trotter Theorem of version holds. Because every line in corresponds to unique point lying on it (vice versa), let this map be , we have
If we assume , then we have , which means is symmetric for and . ∎
Solution 7.9.
Firstly, it’s trivial that there doesn’t exist a component with only one side. If there exist a component with two sides, assuming they’re and , then consider that intersects and at points other than . Such a line must exist, because if not, then every line passes . Notice that intersects area , we know isn’t connected, which leads a contradiction.
Let be the number of vertices on exactly lines and be the number of faces with exactly sides. Notice that and , if , we have . Now
but we know by Euler’s formula. So that leads a contradiction, which means .
Using the map in the previous question, we can get the claim quickly:
Claim. Suppose that is a finite set of points in , not all lying on one line. Then, there is a line that contains exactly two points of .
Just use the conclusion above for , we know there exist a point on exactly two lines and . Then line contains exact two points and . The claim is true for , so the claim is of course true for . ∎
Solution 7.10.
Because the are chosen generically, the values of and values of are all distinct. So the number of points in is . Let denote the unit circle centered at . Then every point with form is on circle , which implies is 3-rich. Thus the number of 3-rich point of is at least . ∎
Solution 7.11.
For every point such that and , it lies on for every . So is a -rich point and . By Theorem 7.1, we know
then we have , which means .
Regarding of the case , let be the set of lines with and . For every point such that and , it lies on for every . So is a -rich point and . By Theorem 7.1, we know
then we have , which means . ∎
Solution 7.12.
Consider the simple graph corresponding to , which has the same vertices as . Every edge of (regardless of the number of repetitions) corresponds to a single edge in . Let be the number of edges in . We have . Thus there is . Using Theorem 7.6 and we know . Notice that , we get . ∎
Solution 7.13.
Let denote the set of points. Assuming , let be vertices of and we consider the circles centered at points in with radii in . The number of circles . Let the edges of be the arcs of circles between consecutive points, for the circles containing more than one point. We don’t construct an edge for the circles containing only one point to avoid loops.
Notice that, for every circle with more than one point lying on it, the number of arcs equals to the points. Let be the circles with only one point. Then . Because there are up to 99 points on the perpendicular bisector of every two points, the number of parallel edges between two points is at most 198, which means .
Firstly, consider the case of , we can use Proposition 7.13 and get . In addition, there is . Therefore, we have
Assume and , then
which means
Thus, we have , that is, . We call as .
For the case of , we have , which is equivalent to . When , we know . When , because , is established. All in all, we have . ∎
Solution 7.14.
Let be vertices of and we consider the unit circles centered at points in . Let the edges of be the arcs of circles between consecutive points, for the circles containing more than one point. Let denote the number of points on the unit circle centered at and denote the number of unit distances. Then we have and . Further, we have
Because two unit circles intersect in at most two points, we have . When , we know , which means . So there is . When , we also have . Thus the conclusion is established. ∎
Solution 7.15.
Use the following method to construct the graph : Let be vertices of . For every curve , we can consider , the set of points on . Let the edges of be the arcs of curves between consecutive points, for the every curve with .
Assuming that is the number of edges in on and , for , we know if closed and if open. Because any two points lie in at most curves of , we get .
Notice that every point is -rich, we have . And notice that , we have
Because any two curves of intersect in at most points, we know .
If , we get , which means . By , we know . Let denote . For the case of , we know , that is,
For the case of , we have .
If , from , we know . For the case of , we have . For the case of , by , we have
In summary, we get . ∎
Solution 7.16.
44 4 The condition in this problem should be , or we have and .Consider the set of parabolas with and set of points with and . Because every satisfies for , we know every point in is -rich point, with . So the conclusion is established. ∎
Solution 7.17.
Firstly, we will show this conclusion.
Claim. Curve intersects in at most 2 points, assuming .
WLOG, we can assume that with . Choose and as the left and right endpoint of (that is, the points on with the smallest and largest coordinate). Let and . Then we have and . 55 5 This part of the proof comes from [5].Since , graphs of and are disjoint, the graphs of are disjoint and the graphs of are also disjoint. Only the graphs of and can intersect. Then, the conclusion reduces to the claim that the graph of a strictly convex function can intersect the graph of a strictly concave function in at most two points. Notice that is strictly convex, we know it has at most 2 roots, so the claim is established.
If there are 3 curves all passing , then passes and . That means intersects at 3 points, which leads a contradiction. So any two points in the plane lie in at most two of the curves of .
Now applying Theorem 7.16, we know . Suppose that has diameter of , assuming , we choose a square such that . We call as and as the number of curves in passing , assuming . Consider and with , we know for every point ,
which means every is contained by curves in . Notice that and , we know
by the inequality above. So we have .
For the case of , we know because . For the case of , we have , so there is . Anyway, the conclusion is established. ∎
Solution 7.18.
We’ll prove this claim firstly.
Claim. For every with satisfying that between any two vertices of , the number of edges is either or lies in , its crossing number .
Consider the simple graph corresponding to , we have , which implies . Take a random subgraph consisting of one edge between each pair of vertices. In the induced embedding on the subgraph, each crossing occurs with probability , since it occurs if and only if both edges are in . Now we have , that is,
since and .
For , let be the subgraph of , satisfying that between any two vertices of , the number of edges is either or lies in . Define and , from , we know
Notice that
by Holder inequality, we know
so we have . ∎
Solution 7.19.
We let be the number of points on circle . Let denote the number of circles containing point . Now we know
Let be the number of circles in , only containing and be the number of circles in , centered at . Notice , we have
So
for . ∎
Solution 7.20.
77 7 Besides hint, this idea also comes from [9].We assume , otherwise we have trivially. By Lemma 7.19, the number of edges of with multiplicity at least is at most . We take , then the value above changed into . Using the bound , we know
when sufficient large. Therefore we have when sufficient large. Now apply Theorem 7.17 on and we know
Using , we know
Thus we have , which means . That is, we have . ∎
Solution 8.1.
88 8 Hint of this problem may has defect. This solution has fixed that.For any point , with , we define a map as follows. If , then we define so that , and are collinear. Now observe that is -rich if and only if . Construct a vertical line perpendicular to plane through point , assume intersects and at respectively. We can get is similar to , which means . Let and , we have . We let , where , then is -rich if and only if . Let’s consider , where are both positive integers satisfying and . Now and .
Claim. Assuming , then for , the number of satisfying is at least .
For , we know , for , where is the number-theoretic inverse of modulo . Notice can take values, we know the claim is established.
When is fixed, let . For every pair, which satisfies and , contribute choices for pairs. (Because , we know , which means , we can use the claim above.) And for every fixed , there are choices of , for and . Now the number of lines passing is .
Then we know
when . Since , we know the inequality above is true. ∎
Solution 8.2.
Step 1: For every , it satisfies and . Then we know
which means .
Step 2: If , we know there exist satisfying for and .
Step 3: Notice that and , thus . We can know is a line from the equation of plane. We call this line as .
Step 4: From doesn’t contain any line, we know also doesn’t contain any line. Since , we consider the points in . It can be realized as finding roots of a quadratic equation whose coefficients are not all 0. (Specifically, we substitute into the equation to solve for .) Thus we know , which leads the conclusion.
Step 5: Suppose that contains the line parametrized by , where and . This is equivalent to saying that for all . If we expand the left-hand side as a polynomial in , each coefficient must vanish, leading to the equations
These formulas lead to a tricky way of writing as a negative square. Since is a quadratic residue, we can conclude that is a quadratic residue.
If , we can know by calculation. So the conclusion is established.
Step 6: If are any two quadratic non-residues, we have
which means is a quadratic residue. So there exists such that . Consider bijection , it maps the element in to since for every , we have
Therefore we have .
If are both non-zero quadratic residues, assuming and , we can consider bijection . Then we know similarly.
For the case of , we consider the equation of . Using the formula in [6]:
where represents the number of roots of in . If , we have . If , we have . Adding them up and we get .
For the case of , we consider the equation of . If , we know . If , we know . Adding them up and we get .
For the case of is quadratic non-residue, we consider the equation of . For every , we have . Since , we know . ∎
Solution 8.3.
Consider the equation of quadratic surface: , where is a symmetric matrix and is a vector. Since the scalar equivalence, we know its degrees of freedom can be realized as a 9-dimension projective space. Every line can be parameterized as . So for and every , which means the coefficients of a quadratic polynomial in are all 0. That is, for and every ,
There are 9 linear equations. Then the dimension of solution is 0, in the sense of scalar equivalence. So there is only one degree 2 algebraic surface containing the three lines. ∎
Solution 8.4.
About the smoothness of , we consider the singularity such that and . Assuming is a line passing , since the two conditions above, we have
Let , here , we know , which means the line passing is on . Since the conclusion above is established for any , we know is a conical surface with vertex . Then won’t be skew because they all passes , which leads a contradiction.
For any point , assume that line passes it. Using Taylor expansion, we know . Now we have and . Equation shows is on tangent plane passing . Notice that reguli are irreducible, we know is not always 0 in that tangent plane. (Otherwise, any will match the condition above, which means vanish in that tangent plane, leading that is reducible.) Since is quadratic, it has at most two linear factors. That is, there are at most two lines satisfy the condition above.
For other fields, the conclusions are also true. For , the proof above also works.. When , the proof should be modified a bit.
Regarding of the smoothness, if there is a singularity, we know . Choose , we have and . Let , then
Expand it and use and to simplify, we have , which means . Because it is established for any , we know contains a plane, which is contradictory to that is irreducible.
Regarding that there are at most two lines in passing one point, just expand and simplify. We can get , which means and if for any . And that’s contradictory to that is irreducible. ∎
Solution 8.5.
With a similar proof of Theorem 8.18, we can know:
Claim. Suppose that is a set of lines in with lines in any plane or degree 2 surface. Then the number of intersection points of is , where is constant .
Just replace all ”10” of the proof of Theorem 8.18 with ””, we can finish proof of the claim above.
We will prove by induction that a set of lines determines at most joints. More precisely, here .
Take , let , now . If , we have and . So the inductive foundation is established. We can assume that, for any containing lines, we have . Now consider the line set with lines.
For the case that there is no plane or regulus containing more than lines, consider the number of intersections, we have .
If there exist a plane or regulus containing more than lines, we call it and let denote the lines in . Notice that , let and we have . If is plane, any three lines are coplanar,then there doesn’t exist joint on . If is regulus, any point lies on at most two lines, then there doesn’t exist joint on it. So there’s no joint in .
The joints in can be divided into two categories. In the first category, the three lines that make up the joints are all in , the number of these joints is . In the second category, those three lines have at least one in and at least one in . For this category, every line in intersects at most 2 points. So the number of joints in the second category is at most . Now we have
The last inequality holds when and , here we omit some of the calculation details. So the induction finishes and conclusion is established. ∎
Solution 10.1.
Let be a degree that we will choose later. By Theorem 10.3, we can find a non-zero polynomial of degree so that each component of the complement of contains points of . Let be the components of . For each , let and let be the set of unit circles of that intersect . Let and . Since circle is a quadratic curve, if it isn’t contained in , then it intersects in at most points, by Bezout Theorem. Thus every unit circle passes cells, which means .
Claim. If is a set of points in , and is a set of unit circles in , then
(1) ;
(2) .
Fix . Let be the number of circles of that contain and no other point of . For each other point , there is at most two circles of containing and . Therefore, . So . The proof of the other inequality is similar.
If or , then the conclusion follows from the claim above. Therefore, we can now restrict to the case that .
Applying the claim in each cell, we get then
Since every circle intersects in at most points if it isn’t contained in , we have . Consider the degree, the number of circles in is at most . Using the claim above, we get .
Adding them up and we know . Take , from , we know . Also from , we know . Therefore, . And let be the unit circles centered at , we know and , which means a set of points in the plane determines unit distances. ∎
Solution 10.2.
99 9 Conclusions needed to prove should be and .Consider the circumcenter of a triangle, we can know a set of 3 points lies on a unique circle immediately. Fix . Let be the circles containing at least 3 points in and . Therefore, . So .
Let be a degree that we will choose later. By Theorem 10.3, we can find a non-zero polynomial of degree so that each component of the complement of contains points of . Let be the components of . For each , let and let be the set of circles of that intersect . Let and . Since circle is a quadratic curve, if it isn’t contained in , then it intersects in at most points, by Bezout Theorem. Thus every circle passes cells, which means .
Applying that in each cell, we get
Since every circle intersects in at most points if it isn’t contained in , we have . Consider the degree, the number of circles in is at most .
We can prove by the method in Exercise 10.1. Using the inequality above, we get .
If or , then we know or respectively. Therefore, we can now restrict to the case that .
Adding them up and we know . Take , from , we know . Also from , we know . Therefore, .
Regarding of the -rich point. Consider the as , we have . For the case of , we have and . Then we have immediately.1010 10 If , the conclusion may be not established. Consider an counterexample: there is a point lying on circles and we have , but let we have .
∎
Solution 10.3.
Suppose that is a set of parabolas in the plane and is a set of points in the plane. Here is an example of . Take , we assume as a large integer. Notice that for every integer such that , we have since . Now we have and . Thus we have . ∎
Solution 10.4.
First, if there exist two irreducible algebraic curves and with degree at most , the number of intersections of them is at most , since Bezout Theorem. So for any set of points in , there is at most one curve containing all the points.
Fix . Let be the irreducible algebraic curves containing at least points in and . Therefore, . So .
Let be a degree that we will choose later. By Theorem 10.3, we can find a non-zero polynomial of degree so that each component of the complement of contains points of . Let be the components of . For each , let and let be the set of irreducible algebraic curves of that intersect . Let and . Since the degree of irreducible algebraic curves is at most , if it isn’t contained in , then it intersects in at most points, by Bezout Theorem. Thus every irreducible algebraic curve passes cells, which means .
Applying that in each cell, we get
Since every irreducible algebraic curve intersects in at most points if it isn’t contained in , we have . Consider the degree, the number of irreducible algebraic curves in is at most .
We can prove by the method in Exercise 10.1. Using the inequality above, we get .
If or , then we know or respectively. Therefore, we can now restrict to the case that .
Adding them up and we know . Take , from , we know . Also from , we know . Therefore, . ∎
Solution 10.5.
We will use induction to prove the following claim firstly.
Claim. For any , if is a set of lines in the plane, and is a set of points in the plane, then .
We can observe that the conclusion is established when . Assuming that the conclusion is true for such that .
By Theorem 10.3, we can find a non-zero polynomial of degree so that each component of the complement of contains points of , where is a constant. Let is the number of cells and be the components of . For each , let and let be the set of lines of that intersect . Let and . Since every line intersects in at most points, then every line passes cells, which means .
If or , then we know or respectively. Therefore, we can now restrict to the case that .
In following content, we assume that
which are possible when and are both large, where .
Applying that in each cell, using Jensen’s inequality, we get
Also, similar to the proof of Theorem 10.10, we know and . Then we know
since and . Now adding them up and we finish the proof of this claim.
Notice that , we have
∎
Solution 10.6.
Fix . Let be the number of planes of that contain and no other point of . For each other point , there is at most two planes of containing and . Therefore, . So .
Fix . Let be the points contained in least 3 planes in and . Therefore, . So .
Claim. For any , if is a set of planes in , and is a set of points in , then .
We can observe that the conclusion is established when . Assuming that the conclusion is true for such that .
By Theorem 10.3, we can find a non-zero polynomial of degree so that each component of the complement of contains points of , where is a constant. Let is the number of cells and be the components of . For each , let and let be the set of planes of that intersect . Let and . According to Harnack inequality, every plane intersects in at most cells, which means , where is a constant. Assume is the set of planes of that lie in and .
Let be the set of points of that lie in and the set of points of that lie in . Again by Theorem 10.3, we can find a non-zero polynomial of degree which is co-prime to so that each component of the complement of contains points of , where is a constant. (That is possible because if not, we can consider a small perturbation of to make they are co-prime.) Let is the number of cells and be the components of . For each , let and let be the set of planes of that intersect . Let and . According to Harnack inequality, every plane intersects in at most cells, which means , where is a constant.
If or , then we know or respectively. Therefore, we can now restrict to the case that .
In following content, we assume that
and
which are possible when and are both large, where and .
Applying that in each cell, using Jensen’s inequality, let , we get
Assume , we have
Now we know .
Consider Bezout Theorem, each plane of has at most intersection points with , and so it has at most incidences with , we know . And using the inequality above, we have . Then we know
since and . Now adding them up and we finish the proof of this claim.
Notice that , we have
∎
Solution 10.7.
Considering a circle large enough, which isn’t contained in , we know by Bezout Theorem. Notice that every unbounded component must intersect , we know the number of unbounded components is at most .
Regarding the bounded components, in every bounded component , we know must reach a local maximum or a local minimum, which means there is a critical point in . If there’s no common factor of and , then the number of points such that is at most . Then the number of bounded components is at most .
For the case of and have at least one common factor, we can consider the polynomial . Let , we take and small enough such that for in every bounded component. So for every bounded , there exist a point in it such that . And for , we know . Thus there is a critical point of in . Notice that, for small enough (for all where is a constant depending ), and have no common factor, we know that we can always make and have no common factor. Therefore we know the number of bounded components is at most . The number of components of is at most . ∎
Solution 10.8.
Suppose that there are three unit 2-spheres . Notice that and are both circles and , we know since two circles intersect in at most 2 points.
Fix . Let be the unit 2-spheres containing at least points in and . Notice that 3 points can determine at most two unit 2-spheres. Therefore, . So .
Fix . Let be the points contained in least 3 unit 2-spheres in and . Notice that 3 unit 2-spheres intersect in at most two points. Therefore, . So .
Claim. For any , if is a set of unit 2-spheres in , and is a set of points in , then .
By Theorem 10.3, we can find a non-zero polynomial of degree so that each component of the complement of contains points of , where is a constant. Let is the number of cells and be the components of . For each , let and let be the set of unit 2-spheres of that intersect . Let and . According to spherical version of the Harnack inequality, every unit 2-sphere intersects in at most cells, which means , where is a constant. Assume is the set of unit 2-spheres of that lie in and .
Let be the set of points of that lie in and the set of points of that lie in . Again by Theorem 10.3, we can find a non-zero polynomial of degree which is co-prime to so that each component of the complement of contains points of , where is a constant. (That is possible because if not, we can consider a small perturbation of to make they are co-prime.) Let is the number of cells and be the components of . For each , let and let be the set of unit 2-spheres of that intersect . Let and . According to spherical version of the Harnack inequality, every unit 2-spheres intersects in at most cells, which means , where is a constant.
If or , then we know or respectively. Therefore, we can now restrict to the case that .
In following content, we assume that
and
which are possible when and are both large, where .
Applying that in each cell, using Jensen’s inequality, let , we get
Assume , we have
Now we know .
Consider Bezout Theorem and that sphere is irreducible in , each unit 2-sphere of has at most intersection points with , and so it has at most incidences with , we know . Consider the degree, the number of unit 2-sphere in is at most . And using the inequality above, we have . Then we know
since and . Now adding them up and we finish the proof of this claim.
Notice that , we have
Take as the unit 2-spheres centered at points in and we have . And we can get that points in determine at most unit distances immediately. ∎
Solution 10.9.
Fix . Let be the 2-spheres containing at least points in and . Notice that 11 points can determine a sphere. Therefore, . So .
Fix . Let be the number of points of that contained in and no other sphere of . Notice that two spheres intersect in at most 10 points. Therefore, , so we have . 1212 12 Since here we can use a more accurate estimate of , it is possible that the result of this problem can be better.
Claim. For any , if is a set of 2-spheres in , and is a set of points in , then .
By Theorem 10.3, we can find a non-zero polynomial of degree so that each component of the complement of contains points of , where is a constant. Let is the number of cells and be the components of . For each , let and let be the set of 2-spheres of that intersect . Let and . According to spherical version of the Harnack inequality, every 2-sphere intersects in at most cells, which means , where is a constant. Assume is the set of 2-spheres of that lie in and .
Let be the set of points of that lie in and the set of points of that lie in . Again by Theorem 10.3, we can find a non-zero polynomial of degree which is co-prime to so that each component of the complement of contains points of , where is a constant. (That is possible because if not, we can consider a small perturbation of to make they are co-prime.) Let is the number of cells and be the components of . For each , let and let be the set of 2-spheres of that intersect . Let and . According to spherical version of the Harnack inequality, every 2-spheres intersects in at most cells, which means , where is a constant.
If or , then we know or respectively. Therefore, we can now restrict to the case that .
In following content, we assume that
and
which are possible when and are both large, where and .
Applying that in each cell, using Jensen’s inequality, let , we get
Assume , we have
Now we know .
Consider Bezout Theorem and that sphere is irreducible in , each 2-sphere of has at most intersection points with , and so it has at most incidences with , we know . Consider the degree, the number of unit 2-sphere in is at most . And using the inequality above, we have . Then we know
since and . Now adding them up and we finish the proof of this claim.
Notice that , we have
∎
Solution 10.10.
By Theorem 10.3, there exist a polynomial such that is decomposed into cells, satisfying for every . We have .
For the case of , we know for every , we have , where is set of lines that intersect . Notice that every line intersects at most cells, we have . Since the number of cells is , there exist a cell such that . So there is
For the case of , we know by Lemma 10.15.
Therefore, we know , which means . Take , we get
∎
Solution 10.11.
We will use induction on to prove this conclusion. It obvious that it is established for . We know there is a constant such that . We assume that this conclusion is established for with , that is, .
Suppose that and large enough, satisfying the following four conditions:
Since , we can use the induction hypothesis that
So we have
that is
Use the conditions above, we have
Thus we have , which leads the conclusion. ∎
Solution 10.12.
For the case of , then by Lemma 10.16.
If , notice that Lemma 10.15 is also available for and also sharp, because all lines can be on the same 2-plane contained by . Similar to Exercise 10.10, we can prove that there exists a constant such that
Claim. We have .
We use induction on to prove this conclusion. It obvious that it is established for . We assume that this conclusion is established for with , that is, .
Suppose that and large enough, satisfying the following four conditions:
Since , we can use the induction hypothesis that
So we have
that is
Use the conditions above, we have
Thus we have . So we have . ∎
References
- [1] Degree of image of a polynomial map. Note: MathOverflowURL:https://mathoverflow.net/q/63463 (version: 2011-04-29) External Links: https://mathoverflow.net/q/63463, Link Cited by: Solution 6.2.
- [2] (2014-Dec.) Incidences with -non-degenerate sets and their applications. Journal of Computational Geometry 5 (1), pp. 284–302. External Links: Link, Document Cited by: footnote 11.
- [3] The lebesgue measure of zero set of a polynomial function is zero. Note: Mathematics Stack ExchangeURL:https://math.stackexchange.com/q/1920527 (version: 2021-07-22) External Links: https://math.stackexchange.com/q/1920527, Link Cited by: footnote 1.
- [4] (1977) Varieties. In Algebraic Geometry, pp. 1–59. External Links: ISBN 978-1-4757-3849-0, Document, Link Cited by: Solution 6.1, Solution 6.1.
- [5] And are non-crossing if they are translates of a convex disc. Note: Mathematics Stack ExchangeURL:https://math.stackexchange.com/q/3225611 (version: 2019-05-14) External Links: https://math.stackexchange.com/q/3225611, Link Cited by: footnote 5.
- [6] (1996) Equations over finite fields. In Finite Fields, Encyclopedia of Mathematics and its Applications, pp. 268–346. Cited by: Solution 3.2, Solution 3.2, Solution 8.2.
- [7] Polynomial that can be represented as a linear combination. Note: Mathematics Stack ExchangeURL:https://math.stackexchange.com/q/5051406 (version: 2025-03-30) External Links: https://math.stackexchange.com/q/5051406, Link Cited by: footnote 2.
- [8] (2015) Hilbert functions and the finite degree zariski closure in finite field combinatorial geometry. Journal of Combinatorial Theory, Series A 134, pp. 196–220. External Links: ISSN 0097-3165, Document, Link Cited by: Solution 2.9.
- [9] (1997) Crossing numbers and hard erdos problems in discrete geometry. Combinatorics, Probability and Computing 6 (3), pp. 353–358. External Links: Document Cited by: footnote 6, footnote 7.